Bạn chưa đăng nhập. Vui lòng đăng nhập để hỏi bài

Những câu hỏi liên quan
Trần Tuấn Hoàng
Xem chi tiết
Mai Tiến Đỗ
Xem chi tiết
Nguyễn Việt Lâm
23 tháng 1 2021 lúc 11:18

\(abc\ge\left(a+b-c\right)\left(b+c-a\right)\left(c+a-b\right)\)

\(\Leftrightarrow abc\ge\left(3-2a\right)\left(3-2b\right)\left(3-2c\right)\)

\(\Leftrightarrow9abc\ge12\left(ab+bc+ca\right)-27\)

\(\Rightarrow abc\ge\dfrac{4}{3}\left(ab+bc+ca\right)-3\)

\(P\ge\dfrac{9}{a\left(b^2+bc+c^2\right)+b\left(c^2+ca+a^2\right)+c\left(a^2+ab+b^2\right)}+\dfrac{abc}{ab+bc+ca}=\dfrac{9}{\left(ab+bc+ca\right)\left(a+b+c\right)}+\dfrac{abc}{ab+bc+ca}\)

\(\Rightarrow P\ge\dfrac{3}{ab+bc+ca}+\dfrac{abc}{ab+bc+ca}=\dfrac{3+abc}{ab+bc+ca}\)

\(\Rightarrow P\ge\dfrac{3+\dfrac{4}{3}\left(ab+bc+ca\right)-3}{ab+bc+ca}=\dfrac{4}{3}\)

Dấu "=" xảy ra khi \(a=b=c=1\)

VUX NA
Xem chi tiết
Nguyễn Việt Lâm
21 tháng 8 2021 lúc 16:11

\(Q=\sum\dfrac{\left(a+b\right)^2}{\sqrt{2\left(b+c\right)^2+bc}}\ge\sum\dfrac{\left(a+b\right)^2}{\sqrt{2\left(b+c\right)^2+\dfrac{1}{4}\left(b+c\right)^2}}=\dfrac{2}{3}\sum\dfrac{\left(a+b\right)^2}{b+c}\)

\(Q\ge\dfrac{2}{3}.\dfrac{\left(a+b+b+c+c+a\right)^2}{a+b+b+c+c+a}=\dfrac{4}{3}\left(a+b+c\right)=\dfrac{4}{3}\)

VUX NA
Xem chi tiết
Nguyễn Việt Lâm
17 tháng 3 2022 lúc 21:48

\(P\ge3\sqrt[3]{\dfrac{abc\left(a^2+1\right)^2\left(b^2+1\right)^2\left(c^2+1\right)^2}{a^2b^2c^2\left(a^2+1\right)\left(b^2+1\right)\left(c^2+1\right)}}=3\sqrt[3]{\dfrac{\left(a^2+1\right)\left(b^2+1\right)\left(c^2+1\right)}{abc}}\)

\(P\ge3\sqrt[3]{\dfrac{\left(a^2+1\right)\left(b^2+1\right)\left(c^2+1\right)}{\left(\dfrac{a+b+c}{3}\right)^3}}=9\sqrt[3]{\dfrac{\left(a^2+1\right)\left(b^2+1\right)\left(c^2+1\right)}{\left(a+b+c\right)^3}}\ge9\sqrt[3]{\dfrac{\left(a^2+1\right)\left(b^2+1\right)\left(c^2+1\right)}{2\left(a+b+c\right)^2}}\)

Theo nguyên lý Dirichlet, trong 3 số \(a^2;b^2;c^2\) luôn có ít nhất 2 số cùng phía so với \(\dfrac{4}{9}\)

Không mất tính tổng quát, giả sử đó là \(a^2;b^2\)

\(\Rightarrow\left(a^2-\dfrac{4}{9}\right)\left(b^2-\dfrac{4}{9}\right)\ge0\)

\(\Leftrightarrow a^2b^2+\dfrac{16}{81}\ge\dfrac{4}{9}a^2+\dfrac{4}{9}b^2\)

\(\Rightarrow a^2b^2+a^2+b^2+1\ge\dfrac{13}{9}a^2+\dfrac{13}{9}b^2+\dfrac{65}{81}\)

\(\Rightarrow\left(a^2+1\right)\left(b^2+1\right)\ge\dfrac{13}{9}\left(a^2+b^2+\dfrac{5}{9}\right)\)

\(\Rightarrow\left(a^2+1\right)\left(b^2+1\right)\left(c^2+1\right)\ge\dfrac{13}{9}\left(a^2+b^2+\dfrac{5}{9}\right)\left(c^2+1\right)\)

\(=\dfrac{13}{9}\left(a^2+b^2+\dfrac{4}{9}+\dfrac{1}{9}\right)\left(\dfrac{4}{9}+\dfrac{4}{9}+c^2+\dfrac{1}{9}\right)\)

\(\ge\dfrac{13}{9}\left(\dfrac{2}{3}a+\dfrac{2}{3}b+\dfrac{2}{3}c+\dfrac{1}{9}\right)^2\)

\(\Rightarrow P\ge9\sqrt[3]{\dfrac{\dfrac{13}{9}\left(\dfrac{2}{3}\left(a+b+c\right)+\dfrac{1}{9}\right)^2}{2\left(a+b+c\right)^2}}=9\sqrt[3]{\dfrac{13}{18}\left(\dfrac{2}{3}+\dfrac{1}{9\left(a+b+c\right)}\right)^2}\)

\(P\ge9\sqrt[3]{\dfrac{13}{18}\left(\dfrac{2}{3}+\dfrac{1}{9.2}\right)^2}=\dfrac{13}{2}\)

\(P_{min}=\dfrac{13}{2}\) khi \(a=b=c=\dfrac{2}{3}\)

Nguyễn Việt Lâm
19 tháng 3 2022 lúc 15:08

Từ giả thiết \(2\ge a+b+c\ge3\sqrt[3]{abc}\Rightarrow\sqrt[3]{abc}\le\dfrac{2}{3}\)

\(P\ge3\sqrt[3]{\dfrac{\left(a^2+1\right)\left(b^2+1\right)\left(c^2+1\right)}{abc}}\)

Đặt \(Q=\dfrac{\left(a^2+1\right)\left(b^2+1\right)\left(c^2+1\right)}{abc}\)

\(=\dfrac{a^2b^2c^2+\left(a^2b^2+b^2c^2+c^2a^2\right)+\left(a^2+b^2+c^2\right)+1}{abc}\)

\(\ge\dfrac{a^2b^2c^2+3\sqrt[3]{\left(a^2b^2c^2\right)^2}+3\sqrt[3]{a^2b^2c^2}+1}{abc}=\dfrac{\left(\sqrt[3]{a^2b^2c^2}+1\right)^3}{abc}\)

\(=\left(\dfrac{\sqrt[3]{a^2b^2c^2}}{\sqrt[3]{abc}}+\dfrac{1}{\sqrt[3]{abc}}\right)^3=\left(\sqrt[3]{abc}+\dfrac{1}{\sqrt[3]{abc}}\right)^3\)

\(=\left(\sqrt[3]{abc}+\dfrac{4}{9\sqrt[3]{abc}}+\dfrac{5}{9\sqrt[3]{abc}}\right)^3\ge\left(2\sqrt[]{\dfrac{4\sqrt[3]{abc}}{9\sqrt[3]{abc}}}+\dfrac{5}{9.\dfrac{2}{3}}\right)^3=\dfrac{2197}{216}\)

\(\Rightarrow P\ge3\sqrt[3]{\dfrac{2197}{216}}=\dfrac{13}{2}\)

Nguyễn Đức Duy
Xem chi tiết
Nguyễn Thế Hiếu
Xem chi tiết
Nguyễn Việt Lâm
18 tháng 3 2021 lúc 12:31

Ta có:

\(P=\left(\dfrac{1}{a}+\dfrac{1}{b}+\dfrac{1}{c}\right)\left(a+b+c+36abc\right)\)

\(P=\dfrac{a}{b}+\dfrac{b}{a}+\dfrac{b}{c}+\dfrac{c}{b}+\dfrac{a}{c}+\dfrac{c}{a}+3+36\left(ab+bc+ca\right)\)

\(P=\dfrac{a^2+b^2}{ab}+\dfrac{b^2+c^2}{bc}+\dfrac{c^2+a^2}{ca}+3+36\left(ab+bc+ca\right)\)

\(P=\dfrac{\left(a+b\right)^2}{ab}+\dfrac{\left(b+c\right)^2}{bc}+\dfrac{\left(c+a\right)^2}{ca}-3+36\left(ab+bc+ca\right)\)

\(P\ge\dfrac{4\left(a+b+c\right)^2}{ab+bc+ca}-3+36\left(ab+bc+ca\right)\)

\(P\ge\dfrac{4}{ab+bc+ca}+36\left(ab+bc+ca\right)-3\ge2\sqrt{\dfrac{4.36\left(ab+bc+ca\right)}{ab+bc+ca}}-3=21\)

Dấu "=" xảy ra khi \(a=b=c=\dfrac{1}{3}\)

Akai Haruma
18 tháng 3 2021 lúc 2:49

Lời giải:

Nếu bạn học dồn biến- thừa trừ rồi thì có thể làm như sau:

$P=\frac{ab+bc+ac}{abc}(1+36abc)=\frac{1}{a}+\frac{1}{b}+\frac{1}{c}+36(ab+bc+ac)=f(a,b,c)$

Giả sử $c=\max(a,b,c)$. Ta sẽ chứng minh $f(a,b,c)\geq f(\frac{a+b}{2}, \frac{a+b}{2}, c)$

Thật vậy:

\(f(a,b,c)- f(\frac{a+b}{2}, \frac{a+b}{2}, c)=\frac{(a+b)^2-4ab}{ab(a+b)}+36.\frac{4ab-(a+b)^2}{4}\)

\(=\frac{(a-b)^2}{ab(a+b)}-9(a-b)^2=(a-b)^2(\frac{1}{ab(a+b)}-9)\)

Vì $c=\max (a,b,c)$ mà $a+b+c=1\Rightarrow a+b\leq \frac{2}{3}$

$\Rightarrow ab\leq \frac{1}{4}(a+b)^2\leq \frac{1}{9}$

$\Rightarrow \frac{1}{ab(a+b)}\geq \frac{27}{2}$

$\Rightarrow \frac{1}{ab(a+b)}-9>0$

Do đó: $f(a,b,c)\geq f(\frac{a+b}{2}, \frac{a+b}{2}, c)$

Mà:

$f(\frac{a+b}{2}, \frac{a+b}{2}, c)-21=\frac{4}{a+b}+\frac{1}{c}+36[\frac{(a+b)^2}{4}+c(a+b)]-21$

$=\frac{4}{1-c}+\frac{1}{c}+9(1-c)^2+36c(1-c)-21$

$=\frac{3c+1}{c(1-c)}+9(1-c)^2+36c(1-c)-21$

$=(3c-1)^2.\frac{3c^2-3c+1}{c(1-c)}\geq 0$ với mọi $1>c\geq \frac{1}{3}$

Do đó $f(\frac{a+b}{2}, \frac{a+b}{2}, c)\geq 21$

$\Rightarrow f(a,b,c)\geq 21$

Hay $P_{\min}=21$

Nezuko Kamado
Xem chi tiết
Nezuko Kamado
31 tháng 10 2021 lúc 13:35

Ai lm đc câu nào thì giúp mk với , cảm ơn !!

Nguyễn Hoàng Minh
31 tháng 10 2021 lúc 13:39

\(A=\left|\dfrac{3}{5}-x\right|+\dfrac{1}{9}\ge\dfrac{1}{9}\\ A_{min}=\dfrac{1}{9}\Leftrightarrow x=\dfrac{3}{5}\\ B=\dfrac{2009}{2008}-\left|x-\dfrac{3}{5}\right|\le\dfrac{2009}{2008}\\ B_{max}=\dfrac{2009}{2008}\Leftrightarrow x=\dfrac{3}{5}\\ C=-2\left|\dfrac{1}{3}x+4\right|+1\dfrac{2}{3}\le1\dfrac{2}{3}\\ C_{max}=1\dfrac{2}{3}\Leftrightarrow\dfrac{1}{3}x=-4\Leftrightarrow x=-12\)

Nguyễn Lê Phước Thịnh
31 tháng 10 2021 lúc 13:48

a: \(A=\left|\dfrac{3}{5}-x\right|+\dfrac{1}{9}\ge\dfrac{1}{9}\forall x\)

Dấu '=' xảy ra khi \(x=\dfrac{3}{5}\)

Phạm Kim Oanh
Xem chi tiết
Bùi Đức Huy Hoàng
17 tháng 3 2022 lúc 18:19

a) phương trình \(x^3-3x^2+1\) có 3 nghiệm thực phân biệt là a,b,c(đề bài). Áp dụng Định lí Vi-ét cho đa thức bậc 3 ta có:\(\left\{{}\begin{matrix}a+b+c=3\\ab+bc+ac=0\\a.b.c=-1\end{matrix}\right.\)

ta có

      a+b+c=3

<=>\(\left(a+b+c\right)^2=9\)

<=>\(a^2+b^2+c^2+2ab+2bc+2ac=9\)

<=>\(a^2+b^2+c^2=9\)

<=>\(\left(a^2+b^2+c^2\right)^2=81\)

<=>\(a^4+b^4+c^4+2\left(a^2b^2+b^2c^2+a^2c^2\right)=81\)(1)

ta có ab+bc+ac=0

   <=>\(\left(ab+bc+ac\right)^2=0\)

   <=>\(a^2b^2+b^2c^2+a^2c^2+2abc\left(a+b+c\right)=0\)

   <=>\(a^2b^2+b^2c^2+a^2c^2-2.1.3=0\)

   <=>\(a^2b^2+b^2c^2+a^2c^2=6\)(2)

Thay (2) vào (1) ta có \(a^4+b^4+c^4+2.6=81\)

                                <=>\(a^4+b^4+c^4=69\)

Bùi Đức Huy Hoàng
17 tháng 3 2022 lúc 19:11

b) \(\dfrac{a+1}{\left(b+c\right)\left(1-a\right)+1}=\dfrac{a+1}{\left(3-a\right)\left(1-a\right)+1}=\dfrac{a+1}{3+a^2-4a+1}=\dfrac{a+1}{a^2-4a+4}=\dfrac{a+1}{\left(a-2\right)^2}\)

cmtt =>\(B=\dfrac{a+1}{\left(a-2\right)^2}+\dfrac{b+1}{\left(b-2\right)^2}+\dfrac{c+1}{\left(c-2\right)^2}\)=\(\dfrac{1}{a-2}+\dfrac{1}{b-2}+\dfrac{1}{c-2}+3\left[\dfrac{1}{\left(a-2\right)^2}+\dfrac{1}{\left(b-2\right)^2}+\dfrac{1}{\left(c-2\right)^2}\right]\)=\(\dfrac{3\left[\left(a-2\right)\left(b-2\right)\right]^2+3\left[\left(b-2\right)\left(c-a\right)\right]^2+3\left[\left(c-2\right)\left(a-2\right)\right]^2}{\left[\left(a-2\right)\left(b-2\right)\left(c-2\right)\right]^2}\)

đặt t=(a-2)(b-2);u=(b-2)(c-2);v=(c-2)(a-2)     =>t+u+v=0

B thành \(\dfrac{3\left(t^2+u^2+v^2\right)}{t.u.v}\) bạn biến đổi để xuất hiện t+u+v

=>B=\(\dfrac{3\left(t+u+v\right)^2-6\left(t.u+u.v+t.v\right)}{t.u.v}=\dfrac{-6.\left(a-2\right)\left(b-2\right)\left(c-2\right)\left(a-2+b-2+c-2\right)}{t.u.v}=\dfrac{18}{\left(a-2\right)\left(b-2\right)\left(c-2\right)}\)

(a-2)(b-2)(c-2)= abc-2(ab+bc+ac)+4(a+b+c)-8=12-9=3

Vậy B=3

Bùi Đức Huy Hoàng
17 tháng 3 2022 lúc 19:28

c) ta có \(\dfrac{a^3}{a^2+2bc}=\dfrac{a^3}{a^2-2ac-2ab}=\dfrac{a^2}{a-2c-2b}=\dfrac{a^2}{3a-2\left(a+b+c\right)}=\dfrac{a^2}{3\left(a-2\right)}\)

cmtt =>C=\(\dfrac{a^2}{3\left(a-2\right)}+\dfrac{b^2}{3\left(b-2\right)}+\dfrac{c^2}{3\left(c-2\right)}=\dfrac{a^2\left(b-2\right)\left(c-2\right)+b^2\left(a-2\right)\left(c-2\right)+c^2\left(a-2\right)\left(b-2\right)}{3\left(a-2\right)\left(b-2\right)\left(c-2\right)}\)

bạn nhân vô thì ra C=\(\dfrac{4a^2-2a\left(ab+ac\right)-a+4b^2-2b\left(bc+ab\right)-b+4c^2-2c\left(ac+bc\right)-c}{3\left(a-2\right)\left(b-2\right)\left(c-2\right)}=\dfrac{ }{ }4\dfrac{ }{ }=\dfrac{4\left(a^2+b^2+c^2\right)-\left(a+b+c\right)+6abc}{3\left(a-2\right)\left(b-2\right)\left(c-2\right)}=\dfrac{4.9-3-6}{3.3}=\dfrac{27}{9}=3\)

Big City Boy
Xem chi tiết
Khôi Bùi
31 tháng 3 2022 lúc 18:47

\(P=2\Sigma a+\Sigma\dfrac{1}{a}=\Sigma a+\Sigma a+\Sigma\dfrac{1}{a}\ge3.\sqrt[3]{\left(\Sigma a\right)^2.\Sigma\dfrac{1}{a}}\)

\(Q=\left(\Sigma a\right)^2.\Sigma\dfrac{1}{a}=\left(3+2\Sigma ab\right).\Sigma\dfrac{1}{a}=3\Sigma\dfrac{1}{a}+4\Sigma a+2\Sigma\dfrac{ab}{c}\ge3\Sigma\dfrac{1}{a}+6\Sigma a=3\left(\Sigma\dfrac{1}{a}+2\Sigma a\right)=3P\)\(\Rightarrow\)\(P\ge3\sqrt[3]{3P}\)   \(\Leftrightarrow P^3\ge81P\Leftrightarrow P^2\ge81\left(P>0\right)\Leftrightarrow P\ge9\)

" = " \(\Leftrightarrow a=b=c=1\)

 

Hoàng Đình Bảo
31 tháng 3 2022 lúc 22:59

Vì $\large a,b,c \in\mathbb{N^*}$ và $\large a^2+b^2+c^2=3\Rightarrow \left\{\begin{matrix} a<\sqrt{3} & \\ b<\sqrt{3} & \\ c<\sqrt{3} & \end{matrix}\right.$

Ta chứng minh bất đẳng thức phụ sau: 

Với $0 <x<\sqrt{3}$ thì $2x+\frac{1}{x} \ge x^2.\frac{1}{2}+\frac{5}{2}(*)$

Thật vậy $(*)$ $\large \Leftrightarrow (x-2)(x-1)^2 \le0$

Do $\large x<\sqrt{3}\Leftrightarrow x<2\Leftrightarrow (x-2)(x-1)^2<0$ (Luôn đúng)

Do đó bất đẳng thức được chứng minh 

Dấu $"="$ xảy ra khi $x=1$

Trở lại bài toán: 

Áp dụng BĐT $(*)$ ta được:

$\large 2a+\frac{1}{a}+2b+\frac{1}{b}+2c+\frac{1}{c}\ge\frac{1}{2}(a^2+b^2+c^2)+\frac{15}{2}=9$

Do $a^2+b^2+c^2=3$

Vậy $GTNN=9$

Dấu $"="$ xảy ra khi: $a=b=c=1$